Difference between revisions of "2020 AMC 10B Problems/Problem 24"

(Solution 8)
m (Solution 1)
 
(4 intermediate revisions by 2 users not shown)
Line 27: Line 27:
 
This means that for <math>n = 400</math> and <math>n = 2500</math>, the equation will hold without the floor function.
 
This means that for <math>n = 400</math> and <math>n = 2500</math>, the equation will hold without the floor function.
  
Now we can simply check the multiples of 70 around 400 and 2500 in the original equation:
+
Now we can simply check the multiples of 70 around 400 and 2500 in the original equation, which we abbreviate as <math>L=R</math>.
  
For <math>n = 330</math>, left hand side  <math>=19</math> but <math>18^2 < 330 < 19^2</math> so right hand side  <math>=18</math>
+
For <math>n = 330</math>, <math>L=19</math> but <math>18^2 < 330 < 19^2</math> so <math>R=18</math>
  
For <math>n = 400</math>, left hand side <math>=20</math> and right hand side <math>=20</math>
+
For <math>n = 400</math>,  <math>L=20</math> and  <math>R=20</math>
  
For <math>n = 470</math>, left hand side <math>=21</math> and right hand side <math>=21</math>
+
For <math>n = 470</math>,  <math>L=21</math>, <math>R=21</math>
  
For <math>n = 540</math>, left hand side  <math>=22</math> but <math>540 > 23^2</math> so right hand side  <math>=23</math>
+
For <math>n = 540</math>, <math>L=22</math> but <math>540 > 23^2</math> so   <math>R=23</math>
  
 
Now we move to <math>n = 2500</math>
 
Now we move to <math>n = 2500</math>
  
For <math>n = 2430</math>, left hand side <math>=49</math> and <math>49^2 < 2430 < 50^2</math> so right hand side  <math>=49</math>
+
For <math>n = 2430</math>,  <math>L=49</math> and <math>49^2 < 2430 < 50^2</math> so <math>R=49</math>
  
For <math>n = 2360</math>, left hand side <math>=48</math> and <math>48^2 < 2360 < 49^2</math> so right hand side  <math>=48</math>
+
For <math>n = 2360</math>,  <math>L=48</math> and <math>48^2 < 2360 < 49^2</math> so <math>R=48</math>
  
For <math>n = 2290</math>, left hand side <math>=47</math> and <math>47^2 < 2360 < 48^2</math> so right hand side <math>=47</math>
+
For <math>n = 2290</math>,  <math>L=47</math> and <math>47^2 < 2360 < 48^2</math> so  <math>R=47</math>
  
For <math>n = 2220</math>, left hand side  <math>=46</math> but <math>47^2 < 2220</math> so right hand side <math>=47</math>
+
For <math>n = 2220</math>, <math>L=46</math> but <math>47^2 < 2220</math> so  <math>R=47</math>
  
For <math>n = 2500</math>, left hand side <math>=50</math> and right hand side  <math>=50</math>
+
For <math>n = 2500</math>,  <math>L=50</math> and <math>R=50</math>
  
For <math>n = 2570</math>, left hand side  <math>=51</math> but <math>2570 < 51^2</math> so right hand side  <math>=50</math>
+
For <math>n = 2570</math>, <math>L=51</math> but <math>2570 < 51^2</math> so <math>R=50</math>
  
 
Therefore we have 6 total solutions, <math>n = 400, 470, 2290, 2360, 2430, 2500 = \boxed{\textbf{(C) 6}}</math>
 
Therefore we have 6 total solutions, <math>n = 400, 470, 2290, 2360, 2430, 2500 = \boxed{\textbf{(C) 6}}</math>
Line 106: Line 106:
 
~KingRavi
 
~KingRavi
  
==Solution 8 ==
+
==Solution 6 ==
  
 
Set <math>x=\sqrt{n}</math> in the given equation and solve for <math>x</math> to get <math>x^2 = 70 \cdot \lfloor x \rfloor - 1000</math>. Set <math>k = \lfloor x \rfloor \ge 0</math>; since <math>\lfloor x \rfloor^2 \le x^2 < (\lfloor x \rfloor + 1)^2</math>, we get  <cmath>k^2 \le  70k - 1000 < k^2 + 2k + 1.</cmath>
 
Set <math>x=\sqrt{n}</math> in the given equation and solve for <math>x</math> to get <math>x^2 = 70 \cdot \lfloor x \rfloor - 1000</math>. Set <math>k = \lfloor x \rfloor \ge 0</math>; since <math>\lfloor x \rfloor^2 \le x^2 < (\lfloor x \rfloor + 1)^2</math>, we get  <cmath>k^2 \le  70k - 1000 < k^2 + 2k + 1.</cmath>
Line 119: Line 119:
 
~[https://artofproblemsolving.com/wiki/index.php/User:Isabelchen isabelchen]
 
~[https://artofproblemsolving.com/wiki/index.php/User:Isabelchen isabelchen]
  
==Solution 9 ==
+
==Solution 7 ==
  
  
Line 156: Line 156:
 
This time, the inequality will hold for all portions of the parabola that are not on or between the its two roots, which are <math>34 + \sqrt{155}>46</math> and <math>34-\sqrt{155}<22</math> (they are roughly equal, but this is to ensure that we do not miss any solutions).  
 
This time, the inequality will hold for all portions of the parabola that are not on or between the its two roots, which are <math>34 + \sqrt{155}>46</math> and <math>34-\sqrt{155}<22</math> (they are roughly equal, but this is to ensure that we do not miss any solutions).  
  
Notation wise, we see all integers <math>k</math> such that  
+
Notation wise, we need all integers <math>k</math> such that  
  
 
<cmath>k \in \left(20, 50\right) \cap \left(-\infty,34 - \sqrt{155} \right)</cmath>
 
<cmath>k \in \left(20, 50\right) \cap \left(-\infty,34 - \sqrt{155} \right)</cmath>
Line 164: Line 164:
 
For the first one, since our uppoer bound is a little less than <math>22</math>, the <math>k</math> that works is <math>21</math>. For the second, our lower bound is a little more than <math>46</math>, so the <math>k</math> that work are <math>47, 48,</math> and  <math>49</math>.  
 
For the first one, since our uppoer bound is a little less than <math>22</math>, the <math>k</math> that works is <math>21</math>. For the second, our lower bound is a little more than <math>46</math>, so the <math>k</math> that work are <math>47, 48,</math> and  <math>49</math>.  
  
<math>\boxed{\textbf{(C) }6}</math> total solutions for <math>n</math>, which are <math>400, 2500, 47^2, 48^2,</math> and <math>49^2.</math>
+
<math>\boxed{\textbf{(C) }6}</math> total solutions for <math>n</math>, since each value of <math>k</math> corresponds to exactly one value of <math>n</math>.
  
 
-Benedict T (countmath1)
 
-Benedict T (countmath1)

Latest revision as of 10:26, 9 March 2024

The following problem is from both the 2020 AMC 10B #24 and 2020 AMC 12B #21, so both problems redirect to this page.

Problem

How many positive integers $n$ satisfy \[\dfrac{n+1000}{70} = \lfloor \sqrt{n} \rfloor?\](Recall that $\lfloor x\rfloor$ is the greatest integer not exceeding $x$.)

$\textbf{(A) } 2 \qquad\textbf{(B) } 4 \qquad\textbf{(C) } 6 \qquad\textbf{(D) } 30 \qquad\textbf{(E) } 32$

Solution 1

We can first consider the equation without a floor function:

\[\dfrac{n+1000}{70} = \sqrt{n}\]

Multiplying both sides by 70 and then squaring:

\[n^2 + 2000n + 1000000 = 4900n\]

Moving all terms to the left:

\[n^2 - 2900n + 1000000 = 0\]

Now we can determine the factors:

\[(n-400)(n-2500) = 0\]

This means that for $n = 400$ and $n = 2500$, the equation will hold without the floor function.

Now we can simply check the multiples of 70 around 400 and 2500 in the original equation, which we abbreviate as $L=R$.

For $n = 330$, $L=19$ but $18^2 < 330 < 19^2$ so $R=18$

For $n = 400$, $L=20$ and $R=20$

For $n = 470$, $L=21$, $R=21$

For $n = 540$, $L=22$ but $540 > 23^2$ so $R=23$

Now we move to $n = 2500$

For $n = 2430$, $L=49$ and $49^2 < 2430 < 50^2$ so $R=49$

For $n = 2360$, $L=48$ and $48^2 < 2360 < 49^2$ so $R=48$

For $n = 2290$, $L=47$ and $47^2 < 2360 < 48^2$ so $R=47$

For $n = 2220$, $L=46$ but $47^2 < 2220$ so $R=47$

For $n = 2500$, $L=50$ and $R=50$

For $n = 2570$, $L=51$ but $2570 < 51^2$ so $R=50$

Therefore we have 6 total solutions, $n = 400, 470, 2290, 2360, 2430, 2500 = \boxed{\textbf{(C) 6}}$

Solution 2

This is my first solution here, so please forgive me for any errors.

We are given that \[\frac{n+1000}{70}=\lfloor\sqrt{n}\rfloor\]

$\lfloor\sqrt{n}\rfloor$ must be an integer, which means that $n+1000$ is divisible by $70$. As $1000\equiv 20\pmod{70}$, this means that $n\equiv 50\pmod{70}$, so we can write $n=70k+50$ for $k\in\mathbb{Z}$.

Therefore, \[\frac{n+1000}{70}=\frac{70k+1050}{70}=k+15=\lfloor\sqrt{70k+50}\rfloor\]

Also, we can say that $\sqrt{70k+50}-1 < k+15$ and $k+15\leq\sqrt{70k+50}$

Squaring the second inequality, we get $k^{2}+30k+225\leq70k+50\implies k^{2}-40k+175\leq 0\implies (k-5)(k-35)\leq0\implies 5\leq k\leq 35$.

Similarly solving the first inequality gives us $k < 19-\sqrt{155}$ or $k > 19+\sqrt{155}$

$\sqrt{155}$ is larger than $12$ and smaller than $13$, so instead, we can say $k\leq 6$ or $k\geq 32$.

Combining this with $5\leq k\leq 35$, we get $k=5,6,32,33,34,35$ are all solutions for $k$ that give a valid solution for $n$, meaning that our answer is $\boxed{\textbf{(C) 6}}$. -Solution By Qqqwerw

Solution 3

We start with the given equation\[\dfrac{n+1000}{70} = \lfloor \sqrt{n} \rfloor\]From there, we can start with the general inequality that $\lfloor \sqrt{n} \rfloor \leq \sqrt{n} < \lfloor \sqrt{n} \rfloor + 1$. This means that\[\dfrac{n+1000}{70} \leq \sqrt{n} < \dfrac{n+1070}{70}\]Solving each inequality separately gives us two inequalities:\[n - 70\sqrt{n} +1000 \leq 0 \rightarrow (\sqrt{n}-50)(\sqrt{n}-20)\leq 0 \rightarrow 20\leq \sqrt{n} \leq 50\]\[n-70\sqrt{n}+1070 > 0 \rightarrow \sqrt{n} < 35-\sqrt{155} , \sqrt{n} > 35+\sqrt{155}\]Simplifying and approximating decimals yields 2 solutions for one inequality and 4 for the other. Hence, the answer is $2+4 = \boxed{\textbf{(C) } 6}$.

~Rekt4

Solution 4

Let $n$ be uniquely of the form $n=k^2+r$ where $0 \le r \le 2k \; \bigstar$. Then, \[\frac{k^2+r+1000}{70} = k\] Rearranging and completeing the square gives \[(k-35)^2 + r = 225\] \[\Rightarrow r = (k-20)(50-k)\; \smiley\] This gives us \[(k-35)^2 \le (k-35)^2+r=225 \le (k-35)^2 + 2k\] Solving the left inequality shows that $20 \le k \le 50$. Combing this with the right inequality gives that \[(k-35)^2+r=225 \le (k-35)^2 + 2k \le (k-35)^2+100\] which implies either $k \ge 47$ or $k \le 23$. By directly computing the cases for $k = 20, 21, 22, 23, 47, 48, 49, 50$ using $\smiley$, it follows that only $k = 22, 23$ yield and invalid $r$ from $\bigstar$. Since each $k$ corresponds to one $r$ and thus to one $n$ (from $\smiley$ and the original form), there must be 6 such $n$.


~the_jake314

Solution 5

Since the right-hand-side is an integer, so must be the left-hand-side. Therefore, we must have $n\equiv -20\pmod{70}$; let $n=70j-20$. The given equation becomes\[j+14 = \lfloor \sqrt{70j-20} \rfloor\]

Since $\lfloor x \rfloor \leq x < \lfloor x \rfloor +1$ for all real $x$, we can take $x=\sqrt{70j-20}$ with $\lfloor x \rfloor =j+14$ to get \[j+14 \leq \sqrt{70j-20} < j+15\] We can square the inequality to get\[196+28j+j^{2} \leq 70j-20 < 225 + 30j + j^{2}\] The left inequality simplifies to $(j-36)(j-6) \leq 0$, which yields \[6 \le j \le 36.\] The right inequality simplifies to $(j-20)^2 - 155 > 0$, which yields \[j < 20 - \sqrt{155} < 8 \quad \text{or} \quad j > 20 + \sqrt{155} > 32\]

Solving $j < 8$, and $6 \le j \le 36$, we get $6 \le j < 8$, for $2$ values $j\in \{6, 7\}$.

Solving $j >32$, and $6 \le j \le 36$, we get $32 < j \le 36$, for $4$ values $k\in \{33, \ldots , 36\}$.

Thus, our answer is $2 + 4 = \boxed{\textbf{(C) }6}$


~KingRavi

Solution 6

Set $x=\sqrt{n}$ in the given equation and solve for $x$ to get $x^2 = 70 \cdot \lfloor x \rfloor - 1000$. Set $k = \lfloor x \rfloor \ge 0$; since $\lfloor x \rfloor^2 \le x^2 < (\lfloor x \rfloor + 1)^2$, we get \[k^2 \le  70k - 1000 < k^2 + 2k + 1.\] The left inequality simplifies to $(k-20)(k-50) \le 0$, which yields \[20 \le k \le 50.\] The right inequality simplifies to $(k-34)^2 > 155$, which yields \[k < 34 - \sqrt{155} < 22 \quad \text{or} \quad k > 34 + \sqrt{155} > 46\] Solving $k < 22$, and $20 \le k \le 50$, we get $20 \le k < 22$, for $2$ values $k\in \{20, 21\}$.

Solving $k >46$, and $20 \le k \le 50$, we get $46 < k \le 50$, for $4$ values $k\in \{47, \ldots , 50\}$.

Thus, our answer is $2 + 4 = \boxed{\textbf{(C) }6}$

~isabelchen

Solution 7

If $n$ is a perfect square, we can write $n = k^2$ for a positive integer $k$, so $\lfloor \sqrt{n} \rfloor = \sqrt{n} = k.$ The given equation turns into

\begin{align*} \frac{k^2 + 1000}{70} &= k \\ k^2 - 70k + 1000 &= 0 \\ (k-20)(k-50) &= 0, \end{align*}

so $k = 20$ or $k= 50$, so $n = 400, 2500.$

If $n$ is not square, then we can say that, for a positive integer $k$, we have \begin{align*} k^2 < &n < (k+1)^2 \\ k^2 + 1000 < &n + 1000 = 70\lfloor \sqrt{n} \rfloor = 70k< (k+1)^2 + 1000 \\ k^2 + 1000 < &70k < (k+1)^2 + 1000. \end{align*}

To solve this inequality, we take the intersection of the two solution sets to each of the two inequalities $k^2 + 1000 < 70k$ and $70k < (k+1)^2 + 1000$. To solve the first one, we have

\begin{align*} k^2 - 70k + 1000 &< 0 \\ (k-20)(k-50) &< 0\\ \end{align*} $k\in (20, 50),$ because the portion of the parabola between its two roots will be negative.

The second inequality yields

\begin{align*} 70k &< k^2 + 2k + 1 + 1000 \\ 0 &< k^2 -68k + 1001. \end{align*} This time, the inequality will hold for all portions of the parabola that are not on or between the its two roots, which are $34 + \sqrt{155}>46$ and $34-\sqrt{155}<22$ (they are roughly equal, but this is to ensure that we do not miss any solutions).

Notation wise, we need all integers $k$ such that

\[k \in \left(20, 50\right) \cap \left(-\infty,34 - \sqrt{155} \right)\] or \[k \in \left(20, 50\right) \cap \left(34 + \sqrt{155}, \infty \right).\]

For the first one, since our uppoer bound is a little less than $22$, the $k$ that works is $21$. For the second, our lower bound is a little more than $46$, so the $k$ that work are $47, 48,$ and $49$.

$\boxed{\textbf{(C) }6}$ total solutions for $n$, since each value of $k$ corresponds to exactly one value of $n$.

-Benedict T (countmath1)

Video Solutions

Video Solution 1

On The Spot STEM: https://youtu.be/BEJybl9TLMA

Video Solution 2

https://www.youtube.com/watch?v=VWeioXzQxVA&list=PLLCzevlMcsWNcTZEaxHe8VaccrhubDOlQ&index=9 ~ MathEx

Video Solution 3 by the Beauty of Math

https://youtu.be/4RVYoeiyC4w?t=62

See Also

2020 AMC 10B (ProblemsAnswer KeyResources)
Preceded by
Problem 23
Followed by
Problem 25
1 2 3 4 5 6 7 8 9 10 11 12 13 14 15 16 17 18 19 20 21 22 23 24 25
All AMC 10 Problems and Solutions
2020 AMC 12B (ProblemsAnswer KeyResources)
Preceded by
Problem 20
Followed by
Problem 22
1 2 3 4 5 6 7 8 9 10 11 12 13 14 15 16 17 18 19 20 21 22 23 24 25
All AMC 12 Problems and Solutions

The problems on this page are copyrighted by the Mathematical Association of America's American Mathematics Competitions. AMC logo.png